domingo, 30 de septiembre de 2012

Problema del día. Algebra (30 de septiembre)

Prueba que para cada entero positivo $n$ existe un número de $n$ dígitos múltiplo de $5^n$ y compuesto únicamente de dígitos impares.

19 comentarios:

  1. $\text{Buscamos un entero }k\text{tal que }$
    $10^n\textgreater 5^nk\textgreater 10^{n-1}$
    $Leftrightarrow 2^n\textgreater k\textgreater 2^{n-1}$

    $\text{Vemos que a partir de }n\textgreater 1,n\equiv 25 \mod{100}$

    $\text{Dividimos en cuatro casos:}$

    $k\equiv 1\mod{4}\Rightarrow 5^nk\equiv 25\mod{100}$
    $k\equiv 2\mod{4}\Rightarrow 5^nk\equiv 50\mod{100}$
    $k\equiv 3\mod{4}\Rightarrow 5^nk\equiv 75\mod{100}$
    $k\equiv 4\mod{4}\Rightarrow 5^nk\equiv 0\mod{100}$

    $\Rightarrow k\equiv 3\mod{4}$

    $\text{Luego vemos que a partir de }n>2\text{ si }$
    $n\equiv 1\mod{2}\Rightarrow 5^n\equiv 125\mod{1000}$
    $n\equiv 0\mod{2}\Rightarrow 5^n\equiv 625\mod{1000}$

    $\text{Luego vemos }5^nk\mod{1000}\text{ y concluimos que si:}$

    $5^n\equiv 125\mod{1000}\Rightarrow k\equiv 3\mod{8}$
    $5^n\equiv 625\mod{100}\Rightarrow k\equiv 7\mod{8}$

    $\text{De todo esto sacamos que:}$
    $2^n\textgreater k\textgreater 2^{n-1}$
    $\text{Si }n\equiv 1\mod{2}\Rightarrow k\equiv 3\mod{8}$
    $\text{Si }n\equiv 0\mod{2}\Rightarrow k\equiv 7\mod{8}$

    $\text{Tambien podemos ver que al fijarnos en }5^n\mod{10^x}$
    $\text{podremos acotar }k\mod{2^x}$

    $\text{Es lo que llevo, aun no encuentro como asegurar la existencia de }k$

    ResponderBorrar
    Respuestas
    1. Incompleto.
      Los casos pequeños están bien, pero falta resolverlo en general.

      Borrar
  2. He visto muy poca actividad en este problema (y ya es el tercer día), así que voy a dar una sugerencia:
    Inducción.

    ResponderBorrar
  3. nos damos cuenta que funciona para el 1 nuestra hipotesis es que funciona hasta n y P.D que funciona has n+1 bueno por la hipotesis tenemos que existe un numero Y congruente a 0 mod 5 a la n, entonces existe un numero 5y tal que se congruente a 0 mod 5 a la n+1 y ya que Y tiene puros numeros impares sabemos que las multiplicaciones van a impares por 5 por lo tanto todos los numeros van a ser impares

    ResponderBorrar
    Respuestas
    1. Incompleto.
      Las multiplicaciones por 5 no generan digitos impares. Basta con ver que $(5)(5)=25$, que tiene un digito par.

      Borrar
  4. Este comentario ha sido eliminado por el autor.

    ResponderBorrar
  5. Lo vemos por inducción.
    Caso base: n=1, 5 es múltiplo de $5^1$ y tiene 1 dígito que es impar.
    Supongamos que es cierto para k.
    Hacemos X el número que cumple para k.
    Las posibles congruencias módulo $5^{k+1}$ de X son $0, 5^k, 2(5^k), 3(5^k), 4(5^k)$ porque X es múltiplo de $5^k$ y $5^{k+1}=5(5^k)$, las posibles congruencias de $10^k$ son son las mismas por las mismas razones. Entonces alguno de $10^k,3(10^k),5(10^k),7(10^k),9(10^k)$ tiene cada una de esas congruencias.
    Si X cumple para k, alguno de $X+10^k,X+3(10^k),X+5(10^k),X+7(10^k),X+9(10^k)$ es congruente a $0 \pmod{k+1}$ (porque sin importar la congruencia de X, alguno de esos múltiplos de $10^k$ es congruente a -X).
    Este número tiene k+1 dígitos (porque X tiene k y al sumarle un múltiplo de $10^k$ le agregamos otro), todos son impares (porque todos los de X lo son y le agregamos un impar) y es múltiplo de $5^{k+1}$, por lo tanto si existe para k entonces existe para k+1, como existe para n=1, existe para todos los naturales.

    ResponderBorrar
  6. Hasta ahorita lo que llevo es lo siguiente:
    -Sabemos que todo $5^n$ termina en $25$, intentamos ver un patrón para ver las ultimas dos cifras de un múltiplo de $5^n$:
    $25*1=25$
    $25*2=50$
    $25*3=75$ Cumple
    $25*4=100$
    $25*5=125$
    $25*6=150$
    $25*7=175$ Cumple
    $25*8=200$
    Observamos que $5^n(x\equiv 3\mod 4)$ es la única manera de que el múltiplo tenga de $5^n$ tenga todas sus cifras impares.
    Podemos descomponer el numero como:
    P.D
    $a_1*10^{n-1}+a_2*10^{n-2}+a_3*10^{n-3}...7*10^{1}+a_5*10^{0}\equv 0\mod 5^n$

    $\rightarrow$
    $a_1*(4y-1)(5^{n-1})+a_2*(4y-1)(5^{n-2})+a_3*(4y-1)(5^{n-3}...7*(4y-1)(10^{1})+5*(4y-1)(10^{0})\equv 0\mod 5^n$

    Esto es lo que llevo

    ResponderBorrar
    Respuestas
    1. Incompleto.
      Es verdad que debe ser $\equiv 3(mod 4)$, pero falta una manera de construir los números.

      Borrar
  7. Esto es lo poco que avance...
    P.D. Que para toda $n$ , se cumple que $5^n \mid M$ , con $M=(5^n)k$ compuesto de $n$ digitos impares.
    Con inducción, hacemos nuestra base:
    $n \rightarrow 5^n \mid M$
    Asegurando que esa esa $n$ existe viendo el caso en que $n=1$ :
    $1 \rightarrow 5^1 \mid 5$
    Entonces buscamos un $n+1$ que cumpla:
    $n+1 \rightarrow 5^{n+1} \mid M_1$ , talque $M_1 = (5^{n+1})q$
    $\Rightarrow M_1 \equiv Y \pmod 5^{n+1}$
    $\Rightarrow Y \equiv M_1 \pmod 5^{n+1}$
    Con eso último no se me ocurrio que hacer, asi que ahora uso $M$ :
    $M \equiv X \pmod 5^{n+1}$
    $(5^n)k \equiv X \pmod 5^{n+1}$
    $\Rightarrow X \equiv (5^n)0 , (5^n)1 , (5^n)2 , (5^n)3 , (5^n)4 \pmod 5^{n+1}$ (Alguna de esas congruencias)
    Creo que de eso puede surgir algo útil... pero hasta aquí tengo.

    ResponderBorrar
    Respuestas
    1. Me di cuenta de que todos los $5^n$ acaban en $25$ , con $n>1$
      Entonces el número al que divide, al ser un múltiplo de $25$ va a terminar en $25$ , $50$ , $75$ o $00$ . Pero como debe estar compuesto de dígitos impares, el número por el que se multiplica $5^n$ , es decir $k$ , debe ser $\equiv 3 \pmod 4$ .
      Falta demostrar que con eso, el número tendría puros dígitos impares.

      Borrar
    2. Este comentario ha sido eliminado por el autor.

      Borrar
    3. Incompleto.
      Es verdad que debe ser $\equiv 3(mod 4)$, pero falta una manera de construir los números.

      Borrar
  8. Intento.-
    Vi los casos chicos para encontrar algún patrón:
    Lo siguiente es de la forma:
    Para:$\rightarrow$Múltiplo de:$\rightarrow$Número que cumple:
    $n=1 \rightarrow 5 \rightarrow 5$
    $n=2 \rightarrow 25 \rightarrow 75$
    $n=3 \rightarrow 125 \rightarrow 375$
    $n=4 \rightarrow 625 \rightarrow 9375$
    $n=5 \rightarrow 3125 \rightarrow 59375$
    $n=6 \rightarrow 15625 \rightarrow 35975$
    De aquí sacamos que para toda $n$ existe un número que cumple tal que es el resultado de agregar un dígito impar a la izquierda al número que cumple para $n-1$ para lo cual intentaré inducción utilizando los casos $n=1, n=2$ como casos base.
    Se debe cumplir que:
    $\exists{k}\forall{n}: 5^{n}a_{1}+10^{n}(2k+1)=5^{n+1}a_{2}$ donde $a_1, a_2\in{N}$ y $0\leq{k}\leq{4}$
    hasta aqui llevo

    ResponderBorrar
    Respuestas
    1. Tu idea es la correcta.
      Para terminar, sólo falta decir que, como $10^n$ es múltiplo de $5^n$ pero no de $5^{n+1}$, entonces entre los números $1\cdot10^n$, $3\cdot10^n$, $5\cdot10^n$, $7\cdot10^n$ y $9\cdot10^n$ hay uno que módulo $5^{n+1}$ va a ser congruente a $-5^na_1$.

      Borrar
  9. Este comentario ha sido eliminado por el autor.

    ResponderBorrar
  10. s facil ver que se cumple para los casos n=1,2,3.
    Veo que $5^n$ divide a $10^n$ entonces escribimos el nuemro $5^n$ veo que para n>3 $5^n$ tiene menos de n digitos entonces si le sumamos $(2k+1)*10^n$ a $5^n$ tal que se forme un numero de n digitos eso seguira siendo multiplo de $5^n$ ya que $5^n$ divide a $10^n$.Entonces viendo que el caso n=3 cumple (375 es el numero que cumple) entonces si a ese numero le agrego un digito impar, y asi le hago para toda n se tendria un numero de n digitos impares, solo hay que ver que sea multiplo de $5^n$. Entonces veo que al numero que cumple para n-1 le tengo que sumar $(2k+1)(10^n)$ solo me falta demostrar que existe $(2k+1)(10^n)$ tal que sea congruente
    a -(el numero que cumple para n-1) modulo $5^n$. Eso es lo que llevo.

    ResponderBorrar
    Respuestas
    1. Tu idea es la correcta.
      Para terminar, sólo falta decir que, como $10^n$ es múltiplo de $5^n$ pero no de $5^{n+1}$, entonces entre los números $1\cdot10^n$, $3\cdot10^n$, $5\cdot10^n$, $7\cdot10^n$ y $9\cdot10^n$ hay uno que módulo $5^{n+1}$ va a ser congruente a -(el numero que cumple para n-1) modulo $5^n$.

      Borrar